When Were Two Trains Closest Together?

Click For Summary
Two trains depart from the same station at different times and speeds: one leaves at 10:00 traveling south at 60 km/h, while the other arrives at the station at 11:00 from the west at 45 km/h. The question focuses on determining when the two trains were closest together during their journeys. Clarification was provided that the second train arrives at the station one hour after the first train departs. After adjusting the equation to account for this timing, the correct answer was confirmed.
Delber
Messages
19
Reaction score
0

Homework Statement


A train leaves the station at 10:00 and travels due south at a speed of 60 km/h. Another train has been heading due west at 45 km/h and reaches the same station at 11:00. At what time were the two trains closest together?

Homework Equations


c^{2}=a^{2}+b^{2}

The Attempt at a Solution


The trouble I am having is the wording of the question. I think it means the trains left from two different stations and will arrive at the same time together at one station. So the equation for that scenario is:

f(t) = \sqrt{(60-60t)^{2}+(45t)^{2}}

However I get a range outside the limit of one hour. I think I'm just confused about he wording if someone can clarify it for me. Any help is appreciated.
 
Physics news on Phys.org
Delber said:

Homework Statement


A train leaves the station at 10:00 and travels due south at a speed of 60 km/h. Another train has been heading due west at 45 km/h and reaches the same station at 11:00. At what time were the two trains closest together?

Hi Delber! :smile:

It means train 2 arrives at the same station one hour after train 1 left. :smile:
 
Thanks for the clarification.

So the new equation should be:

f(t)=\sqrt{(45-45t)^{2}+(60t)^{2}}?

Edit: Yep, I get the correct answer in the book. Thanks for the help.
 
Last edited:
Question: A clock's minute hand has length 4 and its hour hand has length 3. What is the distance between the tips at the moment when it is increasing most rapidly?(Putnam Exam Question) Answer: Making assumption that both the hands moves at constant angular velocities, the answer is ## \sqrt{7} .## But don't you think this assumption is somewhat doubtful and wrong?

Similar threads

  • · Replies 20 ·
Replies
20
Views
3K
  • · Replies 2 ·
Replies
2
Views
2K
  • · Replies 1 ·
Replies
1
Views
4K
  • · Replies 5 ·
Replies
5
Views
7K
  • · Replies 4 ·
Replies
4
Views
4K
Replies
1
Views
2K
  • · Replies 8 ·
Replies
8
Views
3K
  • · Replies 36 ·
2
Replies
36
Views
3K
  • · Replies 5 ·
Replies
5
Views
2K
  • · Replies 2 ·
Replies
2
Views
2K